Last visit was: 24 Apr 2024, 05:48 It is currently 24 Apr 2024, 05:48

Close
GMAT Club Daily Prep
Thank you for using the timer - this advanced tool can estimate your performance and suggest more practice questions. We have subscribed you to Daily Prep Questions via email.

Customized
for You

we will pick new questions that match your level based on your Timer History

Track
Your Progress

every week, we’ll send you an estimated GMAT score based on your performance

Practice
Pays

we will pick new questions that match your level based on your Timer History
Not interested in getting valuable practice questions and articles delivered to your email? No problem, unsubscribe here.
Close
Request Expert Reply
Confirm Cancel
SORT BY:
Date
Tags:
Difficulty: 705-805 Levelx   Percent and Interest Problemsx                              
Show Tags
Hide Tags
avatar
Intern
Intern
Joined: 26 Jul 2010
Posts: 22
Own Kudos [?]: 992 [791]
Given Kudos: 8
Send PM
Most Helpful Reply
Tutor
Joined: 16 Oct 2010
Posts: 14817
Own Kudos [?]: 64892 [595]
Given Kudos: 426
Location: Pune, India
Send PM
User avatar
Current Student
Joined: 07 Aug 2011
Status:mba here i come!
Posts: 151
Own Kudos [?]: 1938 [273]
Given Kudos: 48
Send PM
Math Expert
Joined: 02 Sep 2009
Posts: 92901
Own Kudos [?]: 618726 [100]
Given Kudos: 81586
Send PM
A photography dealer ordered 60 Model X cameras to be sold [#permalink]
52
Kudos
44
Bookmarks
Expert Reply
pgmat wrote:
A photography dealer ordered 60 Model X cameras to be sold for $250 each, which represents a 20 percent markup over the dealer’s initial cost for each camera. Of the cameras ordered, 6 were never sold and were returned to the manufacturer for a refund of 50 percent of the dealer's initial cost. What was the dealer's approximate profit or loss as a percent of the dealer’s initial cost for the 60 cameras?

A. 7% loss
B. 13% loss
C. 7% profit
D. 13% profit
E. 15% profit


The total cost = 60*($250/1.2) = 50*250;

The number of cameras sold = 60 - 6 = 54.
The total revenue = $54*250;

The number of cameras returned = 6.
The total refund = $6*(250/1.2)*0.5;

So, the total income = $54*250+ 6*(250/1.2)*0.5

The dealer's approximate profit is \(\frac{54*250+ 6*(\frac{250}{1.2})*0.5-50*250}{50*250}*100=13\%\)

Answer: D.
User avatar
Current Student
Joined: 15 Sep 2012
Status:Done with formalities.. and back..
Posts: 525
Own Kudos [?]: 1187 [79]
Given Kudos: 23
Location: India
Concentration: Strategy, General Management
Schools: Olin - Wash U - Class of 2015
WE:Information Technology (Computer Software)
Send PM
Re: A photography dealer ordered 60 Model X cameras to be sold [#permalink]
52
Kudos
27
Bookmarks
carcass wrote:
A photography dealer ordered 60 Model X cameras to be sold for$250 each, whichrepresents a 20 percent markup over the dealer's initialcost for each camera.
Of the cameras ordered, 6 were never sold and were returned to the manufacturer for a refund of 50 percent of the dealer's initial cost. What was the dealer's approximate profit or loss as a percent of the dealer's initial cost for the 60 cameras?
(A) 7% loss
(B) 13% loss
(C) 7% profit
(D) 13% profit
(E) 15% profit
have fun. is not difficult to conceptualize but is difficult because if you are not organized and clear you are get in trouble.
I would not show the OA but the rules are stringent. either way try on your own :)


actually, if one could visualize this problem properly, it could be solved in less than 30 secs without getting into any dirty calculations.
Here is my 30 sec approach:
Notice that it is actually a wighted ratio problem, where 9 parts earned 20% profit and 1 part earned 50% loss. (54 for profit, 6 for loss).
Hence overall profit/loss = \((9*20 -1*50) /(9+1) = 13\)
Ans D it is!
User avatar
VP
VP
Joined: 02 Jul 2012
Posts: 1011
Own Kudos [?]: 3117 [63]
Given Kudos: 116
Location: India
Concentration: Strategy
GMAT 1: 740 Q49 V42
GPA: 3.8
WE:Engineering (Energy and Utilities)
Send PM
Re: A photography dealer ordered 60 Model X cameras to be sold [#permalink]
53
Kudos
10
Bookmarks
carcass wrote:
A photography dealer ordered 60 Model X cameras to be sold for$250 each, whichrepresents a 20 percent markup over the dealer's initialcost for each camera.
Of the cameras ordered, 6 were never sold and were returned to the manufacturer for a refund of 50 percent of the dealer's initial cost. What was the dealer's approximate profit or loss as a percent of the dealer's initial cost for the 60 cameras?

(A) 7% loss

(B) 13% loss

(C) 7% profit

(D) 13% profit

(E) 15% profit

have fun. is not difficult to conceptualize but is difficult because if you are not organized and clear you are get in trouble.

I would not show the OA but the rules are stringent. either way try on your own :)


Cost to dealer for 60 cameras = \(\frac{250}{1.2}*60\) = $12,500

Revenue for 54 cameras = 54*250 = $13,500

Revenue from 6 cameras = \(\frac{250}{2.4}*6\) = $625

Total Revenue = $14,125

Profit percent = \(\frac{14125-12500}{12500}\) = 13%

Answer is D.

Otherwise, If one were short on time and I had to guess, One can eliminate A,B & C by looking at the problem itself and improve my chances of guessing the right answer to 50% between D & E.

Kudos Please... If my post helped.
Tutor
Joined: 17 Jul 2019
Posts: 1304
Own Kudos [?]: 2285 [59]
Given Kudos: 66
Location: Canada
GMAT 1: 780 Q51 V45
GMAT 2: 780 Q50 V47
GMAT 3: 770 Q50 V45
Send PM
Re: A photography dealer ordered 60 Model X cameras to be sold [#permalink]
45
Kudos
14
Bookmarks
Expert Reply
Video solution with minimum math and maximum reasoning:
Subscribe for more: https://www.youtube.com/QuantReasoning? ... irmation=1
General Discussion
User avatar
Manager
Manager
Joined: 02 Jun 2011
Posts: 95
Own Kudos [?]: 376 [2]
Given Kudos: 11
Send PM
Re: A photography dealer ordered 60 Model X cameras to be sold [#permalink]
2
Kudos
Bunuel wrote:
pgmat wrote:
A photography dealer ordered 60 Model X cameras to be sold for $250 each, which represents a 20 percent markup over the dealer’s initial cost for each camera. Of the cameras ordered, 6 were never sold and were returned to the manufacturer for a refund of 50 percent of the dealer's initial cost. What was the dealer's approximate profit or loss as a percent of the dealer’s initial cost for the 60 cameras?

A. 7% loss
B. 13% loss
C. 7% profit
D. 13% profit
E. 15% profit


Total cost $60*250/1.2=50*250;

# of cameras sold is 60-6=54 total revenue is 54*250;
# of cameras returned is 6 total refund 6*(250/1.2)*0.5;
So, total income 54*250+ 6*(250/1.2)*0.5

The dealer's approximate profit is (54*250+ 6*(250/1.2)*0.5-50*250)/(50*250)*100=13%

Answer: D.


Dear Bunuel,
Ths is a doubt i face with %ages, in ths quest too.
could you pls tell where i m going wrong?
when 20% mark up for initial cost is given , how to calculate it?
250 - 20/100*250 or to take if 120 is 250 then how much is 100?
how have you arrived at 1.2?
Math Expert
Joined: 02 Sep 2009
Posts: 92901
Own Kudos [?]: 618726 [10]
Given Kudos: 81586
Send PM
Re: A photography dealer ordered 60 Model X cameras to be sold [#permalink]
7
Kudos
3
Bookmarks
Expert Reply
kashishh wrote:
Bunuel wrote:
pgmat wrote:
A photography dealer ordered 60 Model X cameras to be sold for $250 each, which represents a 20 percent markup over the dealer’s initial cost for each camera. Of the cameras ordered, 6 were never sold and were returned to the manufacturer for a refund of 50 percent of the dealer's initial cost. What was the dealer's approximate profit or loss as a percent of the dealer’s initial cost for the 60 cameras?

A. 7% loss
B. 13% loss
C. 7% profit
D. 13% profit
E. 15% profit


Total cost 60*$250/1.2=50*250;

# of cameras sold is 60-6=54 total revenue is 54*250;
# of cameras returned is 6 total refund 6*(250/1.2)*0.5;
So, total income 54*250+ 6*(250/1.2)*0.5

The dealer's approximate profit is (54*250+ 6*(250/1.2)*0.5-50*250)/(50*250)*100=13%

Answer: D.


Dear Bunuel,
Ths is a doubt i face with %ages, in ths quest too.
could you pls tell where i m going wrong?
when 20% mark up for initial cost is given , how to calculate it?
250 - 20/100*250 or to take if 120 is 250 then how much is 100?
how have you arrived at 1.2?


If it's given that the selling price is $250 and the markup over the initial cost is 20%, then: {Cost}+0.2{Cost}=1.2*{Cost}={Selling price} --> 1.2*{Cost}=$250 --> {Cost}=$250/1.2.

Hope it's clear.
avatar
Intern
Intern
Joined: 28 Feb 2012
Posts: 14
Own Kudos [?]: 26 [5]
Given Kudos: 3
GMAT 1: 700 Q48 V39
WE:Information Technology (Computer Software)
Send PM
Re: A photography dealer ordered 60 Model X cameras to be sold [#permalink]
5
Kudos
MBAhereIcome wrote:
54 cameras gave 20% profit while 6 incurred 50% loss. or out of 10, 9 gave profit.

suppose total cameras = 10, out of which 9 gave 20% profit
suppose 100 was the cost of each. total profit 100*10=1000

profit from 9 = 20*9 = 180
loss from 1 = 50*1 = 50
total profit = 180-50 = 130

so, 130 profit from a cost of 1000. that is 13% profit.


Your method seems easy, but I kind of got lost with this - 54 cameras gave 20% profit while 6 incurred 50% loss. or out of 10, 9 gave profit.
Could you please explain, how you deduced this?

Thanks.
User avatar
Manager
Manager
Joined: 02 Jun 2011
Posts: 95
Own Kudos [?]: 376 [7]
Given Kudos: 11
Send PM
Re: A photography dealer ordered 60 Model X cameras to be sold [#permalink]
4
Kudos
3
Bookmarks
gmatDeep wrote:
MBAhereIcome wrote:
54 cameras gave 20% profit while 6 incurred 50% loss. or out of 10, 9 gave profit.

suppose total cameras = 10, out of which 9 gave 20% profit
suppose 100 was the cost of each. total profit 100*10=1000

profit from 9 = 20*9 = 180
loss from 1 = 50*1 = 50
total profit = 180-50 = 130

so, 130 profit from a cost of 1000. that is 13% profit.


Your method seems easy, but I kind of got lost with this - 54 cameras gave 20% profit while 6 incurred 50% loss. or out of 10, 9 gave profit.
Could you please explain, how you deduced this?

Thanks.


yess.. 54:6 = 9:1
mark up is of 20% = profit earned will be 20%
CEO
CEO
Joined: 24 Jul 2011
Status: World Rank #4 MBA Admissions Consultant
Posts: 3187
Own Kudos [?]: 1585 [6]
Given Kudos: 33
GMAT 1: 780 Q51 V48
GRE 1: Q170 V170
Send PM
Re: A photography dealer ordered 60 Model X cameras to be sold [#permalink]
5
Kudos
1
Bookmarks
Expert Reply
60 cameras were ordered to be sold at a 20% markup for $250 each
=> Dealer's cost for each camera = $208
=> Cost of all 60 cameras = $60*208

6 cameras were not sold and returned for a refund of 50% of the dealer's cost
=> The amount the dealer got in returns = 50% of 6*208 = $624
The rest of the cameras (60-6 = 54 in number) were sold
=> Revenue made from selling the cameras = $54 * 250

For the 60 cameras, dealer's profit = 100* [54*(250-208) + (624-1248)]/[(60*200)] =100* (14100-12000)/12000 = ~13% (the error arises because 250/1.2 is not exactly 208 but 208.33)

Option (D)
avatar
Intern
Intern
Joined: 16 Nov 2009
Posts: 1
Own Kudos [?]: 15 [15]
Given Kudos: 0
Send PM
Re: A photography dealer ordered 60 Model X cameras to be sold [#permalink]
11
Kudos
4
Bookmarks
Hi,

For those using normal way to solve this problem, I think that we can avoid huge calculation by just replace Cost by letter C and put it in the final formula:
Income: 54*1.2C + 6*0.5C
Cost: 60*C
Then percent =(Income-cost)/cost = (54*1.2C + 6*0.5C - 60*C)/60*C. We can see that we can eliminate C from the fraction --> percent = (54*1.2 + 6*0.5 - 60)/60 = (54*1.2 + 6*0.5)/60 - 1. Here, we can see clearly the weighted average part of the problem or we can simply calculate the fraction to reach the result.
avatar
Intern
Intern
Joined: 22 Oct 2012
Status:K... M. G...
Posts: 23
Own Kudos [?]: 17 [3]
Given Kudos: 118
Concentration: General Management, Leadership
GMAT Date: 08-27-2013
GPA: 3.8
Send PM
Re: A photography dealer ordered 60 Model X cameras to be sold [#permalink]
3
Kudos
Bunuel wrote:
pgmat wrote:
A photography dealer ordered 60 Model X cameras to be sold for $250 each, which represents a 20 percent markup over the dealer’s initial cost for each camera. Of the cameras ordered, 6 were never sold and were returned to the manufacturer for a refund of 50 percent of the dealer's initial cost. What was the dealer's approximate profit or loss as a percent of the dealer’s initial cost for the 60 cameras?

A. 7% loss
B. 13% loss
C. 7% profit
D. 13% profit
E. 15% profit


Total cost 60*($250/1.2)=50*250;

# of cameras sold is 60-6=54 total revenue is 54*250;
# of cameras returned is 6 total refund 6*(250/1.2)*0.5;
So, total income 54*250+ 6*(250/1.2)*0.5

The dealer's approximate profit is (54*250+ 6*(250/1.2)*0.5-50*250)/(50*250)*100=13%

Answer: D.


this must be kind of awkward question but i got no option other than asking you.

20% markup over the dealer’s initial cost for each camera- I made a wrong assumption , but reducing 20%from 250 & got 200 as an initial cost for each. please explain that "Total cost 60*($250/1.2)=50*250;"
Math Expert
Joined: 02 Sep 2009
Posts: 92901
Own Kudos [?]: 618726 [8]
Given Kudos: 81586
Send PM
Re: A photography dealer ordered 60 Model X cameras to be sold [#permalink]
6
Kudos
2
Bookmarks
Expert Reply
FTGNGU wrote:
Bunuel wrote:
pgmat wrote:
A photography dealer ordered 60 Model X cameras to be sold for $250 each, which represents a 20 percent markup over the dealer’s initial cost for each camera. Of the cameras ordered, 6 were never sold and were returned to the manufacturer for a refund of 50 percent of the dealer's initial cost. What was the dealer's approximate profit or loss as a percent of the dealer’s initial cost for the 60 cameras?

A. 7% loss
B. 13% loss
C. 7% profit
D. 13% profit
E. 15% profit


Total cost 60*($250/1.2)=50*250;

# of cameras sold is 60-6=54 total revenue is 54*250;
# of cameras returned is 6 total refund 6*(250/1.2)*0.5;
So, total income 54*250+ 6*(250/1.2)*0.5

The dealer's approximate profit is (54*250+ 6*(250/1.2)*0.5-50*250)/(50*250)*100=13%

Answer: D.


this must be kind of awkward question but i got no option other than asking you.

20% markup over the dealer’s initial cost for each camera- I made a wrong assumption , but reducing 20%from 250 & got 200 as an initial cost for each. please explain that "Total cost 60*($250/1.2)=50*250;"


(Cost per unit) + 0.2*(Cost per unit) = $250
1.2*(Cost per unit) = $250
(Cost per unit) = $250/1.2

Total cost for 60 units = 60*(Cost per unit) = 60*($250/1.2) = 50*250.

Hope it's clear.
avatar
Intern
Intern
Joined: 02 Mar 2010
Posts: 15
Own Kudos [?]: 32 [11]
Given Kudos: 16
Send PM
Re: A photography dealer ordered 60 Model X cameras to be sold [#permalink]
9
Kudos
2
Bookmarks
Because the answers are in percentages, I thought not to worry about the $ amounts and just focus on the relationships:

Let C be the total cost of all 60 cameras. Originally, the dealer thought to sell all these 60 cameras at 20% profit or for (1.2)C.

However, he sold only (60-6)=54 or 90% of cameras at this price. So revenue from these cameras = (0.9)(1.2)C = (1.08)C

For the remaining 10%, he got a refund of 50% of cost or (0.5)C. So total refund = (0.1)(0.5)C = (0.05)C

Therefore, total revenue in terms of original cost = (1.08 + 0.05)C = 1.13C or 13% profit.

So D is the correct ans.
avatar
Intern
Intern
Joined: 09 Jun 2013
Status:1st attempt: Can I do it?
Posts: 1
Own Kudos [?]: 11 [6]
Given Kudos: 0
Location: India
Concentration: Healthcare, Marketing
GMAT Date: 05-30-2014
GPA: 2.99
WE:Pharmaceuticals (Pharmaceuticals and Biotech)
Send PM
Re: A photography dealer ordered 60 Model X cameras to be sold [#permalink]
4
Kudos
2
Bookmarks
{(54*20) + (6*(-50))}/ 60

This gives 13% profit as answer. 20% proft on 54 cameras and 50% loss on 6 cameras. Use weighted averages here. Rest of the information in the question is misguiding.
User avatar
Manager
Manager
Joined: 28 Dec 2013
Posts: 50
Own Kudos [?]: 14 [1]
Given Kudos: 3
Send PM
Re: A photography dealer ordered 60 Model X cameras to be sold [#permalink]
1
Kudos
VeritasPrepKarishma wrote:
pgmat wrote:
A photography dealer ordered 60 Model X cameras to be sold for $250 each, which represents a 20 percent markup over the dealer’s initial cost for each camera. Of the cameras ordered, 6 were never sold and were returned to the manufacturer for a refund of 50 percent of the dealer's initial cost. What was the dealer's approximate profit or loss as a percent of the dealer’s initial cost for the 60 cameras?

A. 7% loss
B. 13% loss
C. 7% profit
D. 13% profit
E. 15% profit


Use weighted avgs for a quick solution:

On 54 cameras, the dealer made a 20% profit and on 6 cameras, he made a 50% loss. Cost Price of 54 cameras:cost price of 6 cameras = 54:6 = 9:1

Avg Profit/Loss = (.2*9 + (-.5)*1)/10 = 1.3/10 = .13 = 13% profit

For more on weighted avgs, check: https://www.gmatclub.com/forum/veritas-prep-resource-links-no-longer-available-399979.html#/2011/03 ... -averages/

why are we dividing by 10?
Tutor
Joined: 16 Oct 2010
Posts: 14817
Own Kudos [?]: 64892 [1]
Given Kudos: 426
Location: Pune, India
Send PM
Re: A photography dealer ordered 60 Model X cameras to be sold [#permalink]
1
Kudos
Expert Reply
sagnik242 wrote:
VeritasPrepKarishma wrote:
pgmat wrote:
A photography dealer ordered 60 Model X cameras to be sold for $250 each, which represents a 20 percent markup over the dealer’s initial cost for each camera. Of the cameras ordered, 6 were never sold and were returned to the manufacturer for a refund of 50 percent of the dealer's initial cost. What was the dealer's approximate profit or loss as a percent of the dealer’s initial cost for the 60 cameras?

A. 7% loss
B. 13% loss
C. 7% profit
D. 13% profit
E. 15% profit


Use weighted avgs for a quick solution:

On 54 cameras, the dealer made a 20% profit and on 6 cameras, he made a 50% loss. Cost Price of 54 cameras:cost price of 6 cameras = 54:6 = 9:1

Avg Profit/Loss = (.2*9 + (-.5)*1)/10 = 1.3/10 = .13 = 13% profit

For more on weighted avgs, check: https://www.gmatclub.com/forum/veritas-prep-resource-links-no-longer-available-399979.html#/2011/03 ... -averages/

why are we dividing by 10?


Weighted Average Formula:

Cavg = (C1*w1 + C2*w2)/(w1 + w2)

C1 and C2 represent the quantity which we want to average so they will be profit/loss here.
C1 = 20% = .2
C2 = -50% (loss) = -.5
The weights, given by w1 and w2, are the cost prices.
Cost Price of 54 cameras:cost price of 6 cameras = 54:6 = 9:1
So w1 =9 and w2 = 1

Avg Profit/Loss = (.2*9 + (-.5)*1)/(9 + 1) = (.2*9 + (-.5)*1)/10

Get more details on this concept from the link given in my post above.
avatar
Intern
Intern
Joined: 18 Apr 2015
Posts: 1
Own Kudos [?]: [0]
Given Kudos: 0
Send PM
Re: A photography dealer ordered 60 Model X cameras to be sold [#permalink]
Suppose: what if I want to calculate the number of the total profit first. How can I get it? I tried to get it by "54^250/1.2. Is this is the correct way? Pls enlighten. Thanks in advance.
GMAT Club Bot
Re: A photography dealer ordered 60 Model X cameras to be sold [#permalink]
 1   2   3   
Moderators:
Math Expert
92901 posts
Senior Moderator - Masters Forum
3137 posts

Powered by phpBB © phpBB Group | Emoji artwork provided by EmojiOne